LSAT and Law School Admissions Forum

Get expert LSAT preparation and law school admissions advice from PowerScore Test Preparation.

 srcline@noctrl.edu
  • Posts: 243
  • Joined: Oct 16, 2015
|
#32619
Hello

So, I wanted to make sure my thought process was correct as I am revising my study habits and going over my old tests.

So I had originally picked C, But after reviewing, this is incorrect b/c the conclusion is that ginkgo may not have directly enhanced memory. However C, says "some chemical substances that reduce stress in mice also at least temporarily impair their memory". So this would strengthen the argument in showing that the gingo may not have directly enhanced memory.

B is correct b/c it directly weakens that the last sentences b/c B challenges that idea that "other studies have found that ginko reduces stress in mice, and instead saying that both sets of mice whether they were given ginko or not exhibited signs of higher than normal stress. So B is that assumption that the argument fails to account for?

Thankyou
Sarah
 Adam Tyson
PowerScore Staff
  • PowerScore Staff
  • Posts: 5153
  • Joined: Apr 14, 2011
|
#32624
I think you are partway there, Sarah. I wouldn't say that C strengthens the argument, but that it has no impact. What are these substances that might impair memory, and did any of these mice get those substances? What if they did - would that make it more likely that ginkgo directly improved recall, rather than indirectly? Would it make it less likely? I don't think we can say without knowing more.

In the last sentence of the stimulus, the author is implying that ginkgo reduced very high stress, and so indirectly helped recall rather than directly doing so. Answer B attacks that implication by telling us that none of these mice had high stress levels to begin with. That takes away the author's only support for the claim that ginkgo may not have had a direct impact on recall, and so weakens that claim. It doesn't prove anything, but it does weaken the argument somewhat, and that's all we need it to do.

B attacks the author's assumption that the mice had high stress levels. I'm not sure if that's what you meant to say or if you had it backwards, but either way you are thinking about the right ideas there.

Keep up the good work!
 LSAT2018
  • Posts: 242
  • Joined: Jan 10, 2018
|
#46772
Why would answer (A) be incorrect? Wouldn't the difference in dosage suggest that the two experiments cannot be properly compared?
 Jon Denning
PowerScore Staff
  • PowerScore Staff
  • Posts: 904
  • Joined: Apr 11, 2011
|
#46983
Hi LSAT - let's take a look at A and see what's wrong with it!

First, the argument in the stimulus is that ginkgo wasn't directly responsible for improved memory, but rather was a cause of something else (lower stress) that in turn helped improve recall. So ginkgo is the cause of a cause, so to speak.

We want to weaken that idea, and there are a few potential ways to go about it.

One, we could somehow try to dispute the facts given, like "ginkgo doesn't reduce stress" or "reducing stress doesn't improve recall." But that's not really the way the LSAT works: it's not about alternative facts, but rather alternative interpretations!

Two, we could try to somehow cut out stress as a factor/cause. In other words show that stress wasn't an issue in the first place. This is the classic "No Cause but Effect still occurs" attack, and it would certainly hurt the author's position.

Three, we could try to introduce an alternate cause for the improved performance that wasn't about ginkgo or stress or even memory, such as the ginkgo mice were given a bunch of maze practice runs, or were pre-selected for having been particularly adept at maze navigating, or something like that. Or, similarly, add another cause somewhere in the middle, like ginkgo is a great source of energy, and the more energy a mice has the more likely it is to solve mazes. Then it's not low stress, but high energy as the cause.

Four, we could perform some sort of data attack on the experimental parameters, and show that both sets of mice somehow received the same amount of ginkgo in their diets (that would remove ginkgo as a factor altogether and hurt the author). This seems unlikely though since the whole experiment as described in the first sentence was to have the ginkgo be the differentiator.

Turns out what we do in B is option #2: if neither set of mice was stressed, then the argument that ginkgo-lowered stress was the cause makes little sense. You can't lower something—thus making it different for two groups—if it isn't there in the first place!

Answer choice A, however, if anything strengthens the argument! When we're told in A that the mice got more than enough ginkgo to reduce mice stress, it supports the author's claim that ginkgo-lowered stress might be the cause: we at least have a situation sufficient to provide the cause according to A.

If you can't tell that it helps, negate it and watch what happens: the doses of ginkgo the mice received were lower than the doses shown to reduce stress in mice. That instantly attacks the author's position—if they didn't get enough ginkgo to lower stress, then lowered stress isn't the cause—and since the negated form proves harmful it's a great sign that the original helps.

So not only is A wrong, it's in fact the opposite of what we want! Be careful!
 lolaSur
  • Posts: 46
  • Joined: Nov 11, 2019
|
#74181
Hi!

For this question, we are asked to weaken the argument. The conclusion and premise are the following:

Conclusion: G may not have directly enhanced memory
Premise: Other studies have found that G reduces stress in mice AND lowering very high stress levels is known to improve recall.

I usually focus on the conclusion for weakening questions but it seems that here answer B weakens the conclusion by attacking the premise. Answer B states that no mouse exhibited higher than normal stress.
User avatar
 KelseyWoods
PowerScore Staff
  • PowerScore Staff
  • Posts: 1079
  • Joined: Jun 26, 2013
|
#74204
Hi lolaSur!

In Weaken questions, you're basically trying to show that the premises do not necessarily prove the conclusion. It's important to focus on the conclusion because that's key to what you're weakening, but oftentimes what you're doing is weakening the relationship between the premises and conclusion.

This argument has a causal component to it. The conclusion says that the gingko may NOT have caused enhanced memory. Why? Because other studies have shown that gingko causes a reduction in stress and reducing very high stress levels causes memory improvements. So basically the author is suggesting that reducing stress levels may have actually caused the improved memory.

Answer choice (B) weakens the causal conclusion that ginkgo did not cause enhanced memory by saying eliminating stress reduction as a potential cause of the improved memory of the mice in the studies. If none of the mice had high stress levels, then stress reduction can't have been the cause of their improved memory. Getting rid of stress reduction as a potential cause of improved memory attacks the conclusion that the gingko was not a direct cause of the enhanced memory.

Causal reasoning is very common in Weaken questions because causal arguments are inherently flawed--it's almost impossible to definitively prove a causal conclusion! To weaken causal conclusions, you can find an alternate cause, show the cause without the effect or the effect without the cause, show that it's the reverse cause and effect, or find some problem with the data.

Hope this helps!

Best,
Kelsey
 deke97
  • Posts: 10
  • Joined: Jul 06, 2020
|
#77982
Hi! I don't understand why E is wrong. Does this answer not show that ginkgo did not have the intended effect on the mice?
 Luke Haqq
PowerScore Staff
  • PowerScore Staff
  • Posts: 742
  • Joined: Apr 26, 2012
|
#78041
Hi deke97!

To see why answer choice (E) is wrong, it's important to begin by locating the conclusion in this stimulus. Here, it's the sentence on the second-to-last line, "However, the ginkgo may not have directly enhanced memory." To confirm that this is the conclusion, we can see if the other sentences explain the "why"--and we can see that the author explains why this is the case. Namely, it doesn't directly enhance memory because it is instead the lesser amount of stress that helps.

Answer choice (E) states, "The mice who received the ginkgo took just as long as the other mice to learn to navigate the maze." If this were true, it would actually seem to strengthen the author's argument. Answer choice (E) is confirming that the ginkgo wasn't the cause behind improved memory. However, this question stem asks us to weaken the argument rather than strengthen it. This is what answer choice (B) does; the author's conclusion is that it is less stress rather than ginkgo directly that helps improve recall, and answer choice (B) weakens this by supposing that none of the mice had higher than normal stress levels. If this were true, it would weaken the author's claim that "lowering very high stress levels" is the cause with the effect of improving recall. If none of these mice in the experiment had high levels of stress, then it probably wasn't having their stress lowered that accounted for the recall. Instead, perhaps it was the ginkgo acting directly. This would weaken the argument that it was stress reduction rather than gingko acting directly which accounted for memory recall in the experiment.
 kells__w
  • Posts: 10
  • Joined: Mar 29, 2021
|
#86787
Hi, I found the wording of the answer choices to be really confusing. I was between E and B but ultimately picked E not because I thought it was right but because I ruled out B due to what I found to be ambiguous wording. I interpreted B to mean that both groups of mice did not exhibit physiological signs of higher-than-normal stress after the experiment in which case the Ginkgo mice would have taken the extract and were measured to have no higher than normal stress signs following. In which case there would be no way to prove they weren't now exhibiting lower levels of higher stress because they took the ginko and that would if anything potentially strengthen the argument. I thought if that were meant to be the right answer they would have qualified it to include "prior to the study" or something to eliminate the other possibility I thought of. Was my assumption completely unwarranted and is there a better way for me to interpret similar answer choices in these cases to not draw assumptions I did?
Thank you in advance!
User avatar
 Ryan Twomey
PowerScore Staff
  • PowerScore Staff
  • Posts: 141
  • Joined: Mar 04, 2021
|
#86815
Hey Kells,

I would say that it is an unwarranted assumption to assume that the answer choice was referring to the mice after the experiment. I don't think we can assume in this case. But regardless this answer would be correct: this would mean the non-greinko group and the greinko group does not have high stress. This would weaken the author's suggestion that greinko was reducing high stress and thus indirectly improving memory, because neither the greinko group nor the non-greinko group had high stress according to answer choice B.

This is a tricky but perfect answer choice attacking the author's main suggestion that greinko reduced high stress and thus indirectly improved memory.

I hope this helps.

Best,
Ryan

Get the most out of your LSAT Prep Plus subscription.

Analyze and track your performance with our Testing and Analytics Package.